Write the expression using exponents.

3 × 3 × 3 × 3

A. 33
B. 34
C. 43
D. 35

Answers

Answer 1

Answer:

B

Step-by-step explanation:


Related Questions

Jayden needed gloves
and bought 2 packages
of gloves. There were 5
pairs in each package.
He added this to the 3
pairs of gloves he
already owned. How
many gloves did Jayden
now own?

Answers

Answer:

Jayden has 13 pairs of gloves.

Step-by-step explanation:

2 packages multiplied by 5 pairs = 10

10 pairs plus 3 pairs he already had

= 13 pairs of gloves total  

Solve for the value of K

Answers

Answer:

k = 5

Step-by-step explanation:

The straight angle is 180º

The right angle is 90º

Therefor

(8k + 1) + 90 + 49 = 180

combine like terms

8k + 140 = 180

Subtract 140 from both sides

8k = 40

Divide both sides by 8

k = 5

Answer:

16.25

Step-by-step explanation:

pls help asap! i will mark brainliest!

Answers

Answer:

19 or 17

Step-by-step explanation:

The product of −2 and a number x is greater than 30. Whats the solution of the inequality.


Answers

Answer:  x < -15

Step-by-step explanation:

(-2)*(x) > 30

x < -15  The inequality reverses direction when a negative number is used in the manipulation (we divided by -2).

Victoria is four years older than her neighbor the sum of their ages is no more than 14 years enter an inequality that can be used to represent the situation in the box where X represents Victoria’s neighbors age

Answers

Answer:

Victoria is 4 years older than her neighbor

Step-by-step explanation:

V = x + 4

x = neighbors age

x + x + 4 < = 14...here is the inequality

2x + 4 < = 14.....this is just a simplified form of the inequality

not sure which one u want to write, because they are both basically the same...one is just simplified a little

2x + 4 < = 14

2x < = 14 - 4

2x < = 10

x < = 10/2

x < = 5.....the oldest age of the neighbor is 5

(Hope this helps can I pls have brainlist (crown☺️)

Answer: 5

Step-by-step explanation:

Let x be the age of Victoria's neighbor.

Victoria's age is 4 years older than her neighbor, so her age can be represented as x + 4.

The sum of their ages is no more than 14 years, which can be written as:

x + (x + 4) ≤ 14

Simplifying the inequality:

2x + 4 ≤ 14

2x ≤ 10

x ≤ 5

Therefore, the inequality that represents this situation is x + (x + 4) ≤ 14, where x is less than or equal to 5.

Is this a function?

Answers

Answer:

No, because if you do the vertical line test, the line will hit that two times, Therefore it is not a function.

Step-by-step explanation:

YES I AM GIVING A BRAINLIST :D
Which of the following systems of equations has exactly one solution?

Answers

Answer:

y=10x-2x ................

Put the following equation of a line into slope-intercept form, simplifying all fractions, 6x+4y=-12

Answers

Answer:

Step-by-step explanation:

6x+4y=-12

subtract 6x from both sides in order to move 6x to the other side.

4y=-12-6x

divide 4 from the whole equation in order to make the 4y to y.

y= -3-6/4x

and to simply

y= -1.5x-3

or y= -3/2x-3

Need help with this one too plzzz

Answers

Answer:

0.83

Step-by-step explanation:

volume = length*height*width

1500=L*30*60

1500=1800L

1500/1800 = 0.83

length = 0.83

4. What are deductions?

A.)the amount of money added to the gross pay for commission-based sales

B.)the amount of money included with tips

C.)the amount of money subtracted from the gross pay earned for things like taxes, medical benefits, and retirement benefits

D.)incentives employees earn for the amount sold

Answers

Answer:

C.) the amount of money subtracted from the gross pay earned for things like taxes, medical benefits, and retirement benefits

Step-by-step explanation:

Answer:

the answer is C.) the amount of money subtracted from the gross pay earned for things like taxes, medical benefits, and retirement benefits

help algebra 2 100 points if you answer right

Answers

the second one is the answer <i think>

[tex]\\ \tt\hookrightarrow (f+g)(x)[/tex]

[tex]\\ \tt\hookrightarrow f(x)+g(x)[/tex]

[tex]\\ \tt\hookrightarrow 4x^2-5x+3x^2+6x-4[/tex]

[tex]\\ \tt\hookrightarrow 7x^2+x-4[/tex]

Option B is correct

1 2/5m-3/5(2/3m+1) pls help me I need to simply it

Answers

In fraction form: -(3/5)

Decimal form: -0.6

Hope this helps!

one third of the result of three times a number that is increased by 12

Answers

Answer:

Step-by-step explanation:

1/3 (3x+12)

1/3×3x+1/3×12

x+4

24.0370085031 as a whole number

Answers

Answer:

Two hundred and forty billions, three hundred and seventy millions, eighty five thousand and thirty one

Step-by-step explanation:

1) For each diagram, decide if y is an increase or a decrease relative to x. Then determine how much it increased or decreased.​

Answers

Answer:

i dont know

Step-by-step explanation:

ben put £1000 into a savings account. the interest rate was compound at 0.65% per year. at the end of x years he had made £53.20 in interest show that x=8

Answers

Answer:

Below.

Step-by-step explanation:

Using PRT/100 = I, we can prove that x = 8.

=> 1000 x 0.65 x 8/100

=> 10 x 0.65 x 8

=> 6.5 x 8

=> £52

Unfortunately, something is wrong with the question. Please re-check the question to make sure it is right. I have tripled checked my answer and it is right.



Let x represent BC. Using either ABC or CAB and trigonometric functions, solve for x to three decimal places in at least three different ways.

Show your work for each way you find x.

Answers

Answer:

Step-by-step explanation:

plato answer

x-2y > -8
x + y >1
A. (-5,3)
B. (0,5)
C. (3,-5)
D. (4,0)

Answers

Answer:

A. (-5,3)

Honestly, that is pretty much it and uh.Yeah

Hurry I will give brainlyest!!!!

What are the factors of this expression?

34xy



Drag all of the factors of the expression into the box.

Factors

3/4 -1 3 4 x y

Answers

34, x, y are factors of this expression.

- BRAINLIEST answerer

Question 4 of 30
Find the domain of the graphed function.

Answers

Answer:

A is the answer

Step-by-step explanation:

x³+2x²-15x, x²-7x+12 and 3x²-27 find h.c.f​

Answers

Answer:

[tex] {x}^{3} + 2 {x}^{2} - 15x \\ x( {x}^{2} + 2x - 15) \\ x( {x}^{2} + 5x - 3x - 15) \\ x(x(x + 5) - 3(x + 5) \\ x((x + 5)(x - 3)) \\ {x}^{2} -7x + - 12 \\ {x}^{2} - 4x + 3x - 12 \\ x(x - 4) + 3(x - 4) \\ (x - 4)(x + 3) \\ 3 {x}^{2} - 27 \\ 3( {x}^{2} - 9) \\ 3((x + 3)(x - 3)) \\ \\ the \: hcf \: = (x + 3)[/tex]

PLS HELP ASAP ILL GIVE BRAINLKEST I PROMISE ITS DUE IN A FEW HOUR

Answers

question 1. is 2

question 2 -13

Answer: Question 2 answer is 4        Question 3 answer is 12

Step-by-step explanation:

Question 2: 42*4=168

43*5=210

Mrs. O has $30 to give to students who are doing their best work so far today she has given 1/3 of the money away how much money does she have left?

Answers

Answer: $10

Step-by-step explanation: if helped pls leave a thanks <3

Answer:

$10 left

Step-by-step explanation:

1/3*30=10

logb(x-1) + logb(x+2) = logb(8-2x)

Answers

Answer:

x=2

Step-by-step explanation:

log_x(y)+log_x(z)=log_x(yz), so logb(x-1) + logb(x+2)=logb((x-1)(x+2)). next subtract logb(8-2x) from both sides to get logb((x-1)(x+2))-logb(8-2x)=0. log_x(y) - log_x(z) = log_x(y/z). so now we have logb((x-1)(x+2)/(8-2x)). now you can put it into exponential form: (x-1)(x+2)/(8-2x)=b^0=1 now just solve for x:

(x-1)(x+2)= 8-2x, x^2 + x -2 = 8-2x, x^2 + 3x -10 = 0, (x+5)(x-2)=0 x=-5, x=2. plug both into the original equation to check which one is correct, since log_x(y) can't have a negative y, x=-5 doesnt work

Jada's grandparents started a savings account for her in 2010. The table shows the
amount in the account each year.
year
amount in dollars
2010
600
2012
750
2014
900
2016
1050
If this relationship is graphed with the year on the horizontal axis and the amount in
dollars on the vertical axis, what is the vertical intercept?

Answers

I need help on this one tooo

Solve. 10−2x=3x+7
x=3/5
x=17/5
x = 17

Answers

Answer: x = (3/5)

Step-by-step explanation:

10−2x=3x+7

3 = 5x   [Consolidate terms]

x = (3/5)

evaluate the function h(x) = |-2x|-9 for the given values of x a) h(4)=. b) h(-5)=. c) h(0) =

Answers

Answer:

h(4) = -1 ;   h(-5) = 1        h(0) = -9

Step-by-step explanation:

a)    h(x) = |-2x| - 9       Substitute x = 4

       h(4) = | -2 · 4| - 9

       h(x) = | -8| - 9

       h(4) = 8 - 9

       h(4) = -1

b)  h(x) = |-2x| - 9            Substitute x = -5

    h(-5) = |-2 · -5| - 9

    h(-5) = |-10| - 9

     h(-5) = 10 - 9

      h(-5) = 1

c)   h(x) = |-2x| - 9             Substitute x = 0

     h(0) = |-2 · 0| - 9

     h(0) = |0| - 9

     h(0) = 0 - 9

      h(0) = -9

     

Which line is perpendicular to the line x =
į
?
O A. X = -2y
B. y = - 2x
O
C. X = -2
O
D. y = 2

Answers

D is the correct answer

find the length of x in the figure below. round your answer to 3 significant figure

Answers

Answer:

where is the figure? or tell us the area and width of the figure

Tentukan Penyelesaian dari persamaan berikut

Answers

1. 3z + 11 = -28
3z = -39
z = -13

2. 3a - 1 = a + 9
3a - a = 10
2a = 10
a = 5

3. 2 (3p - 4) = 4p + 6
6p - 8 = 4p + 6
6p - 4p = 14
2p = 14
p = 7

4. 1/2x + 4 = 6
1/2x = 2
x = 2 * 2
x = 4

5. 2/3(6x - 9) = 1/4(4x + 12)
4x - 6 = x + 3
4x - x = 9
3x = 9
x = 3
Other Questions
consider the following :[tex]f(x) = {x}^{2} - x + 2 \div x - 2 = x + 1 + 4 \div x - 2[/tex]which polynomials do you think we should call the divisor,dividend,quotient and remainder ? why doesn't the narrator feel like she's 11 by sandra cisneros Nine more than the quotient of two and a number xxx. PLEASE HELP ME FAST Which of the following best describes the cell below plant cell Protist cell Eukaryotic cellProkaryotic cell The production of orchids by cutting is an example of. Choose the answer that best completes the sentence. 1. The author of this myth is (1 point) O software specialist Glenn Welker. O an elder in the Wasco Indian tribe. O someone from a Pacific Indian tribe. O anonymous." Choose the answer that best completes the sentence. 2. In the days of the animal people, a great creature named Thunderbird lived in the land of the (1 point) O mountains. Orivers. O setting sun. O Thunderbird Alley. Choose the answer that best completes the sentence. 3. Spirit Chief told Coyote to go to the for advice. (1 point) O Above-World O Below-World O Land of the Beyond O highest of the Five Mountains What do we sometimes call the method of holding an item that you are cutting with your non-dominant hand in order to avoid cutting off your fingertips? helping hand cupping suction grip bear claw. y=3x+5 what are the missing valuesA.) ONLY (2,11)B.) ONLY (3,13)C.) BOTH (2,11) and (3,13)D.) NEITHER The pie chart shows information about the voters in an election 2800 more women voted than men work out the total number of voters no link no bot please right fast but right All the assignment for 25 points how has the Internet has enabled the growth of graphic communications? Sub-Saharan Africa is studied as a region because __________. A. most people speak the same language B. most people belong to the same ethnic group C. it is isolated from North Africa by the Sahara Desert D. most people practice the same religion An individual with androgen insensitivity syndrome will:A. Not be feminized at puberty B. Prefer a female sexual partner C. Not function sexually as an adult D. Have female external genitalia A cylinder has a volume of 33,912 units3 and aheight of 12 units. What is the diameter of thecylinder? how to improve your eyesight when you have glasses What is 579625 show working Rosa is a 27-year-old Latina studying at Cal State Dominguez Hills, who is from San Antonio, Texas. She has fallen in love with Andre, a 24-year-old African American who lives in South L.A. and goes to CSU Los Angeles. The two met at a rally against college tuition hikes about 2 years ago, and began dating. They have similar long-term goals and visions for their future, and are ready to be married. However, both Rosas and Andres families are dead set against the marriage, and each family is threatening their family member in order to persuade them not to marry each other. One of the families is even resorting to physical threats. who ever answers the question first will be marked brainliest One pound of ground beef is $3.09. How much is 4.5 pounds? Round to the nearest hundredth, if necessary